Difference between revisions of "2017 AMC 10A Problems/Problem 1"

m
(Solution 6 (quickest))
 
(20 intermediate revisions by 11 users not shown)
Line 1: Line 1:
==Problem==
+
== Problem ==
  
 
What is the value of <math>(2(2(2(2(2(2+1)+1)+1)+1)+1)+1)</math>?
 
What is the value of <math>(2(2(2(2(2(2+1)+1)+1)+1)+1)+1)</math>?
Line 5: Line 5:
 
<math>\textbf{(A)}\ 70\qquad\textbf{(B)}\ 97\qquad\textbf{(C)}\ 127\qquad\textbf{(D)}\ 159\qquad\textbf{(E)}\ 729</math>
 
<math>\textbf{(A)}\ 70\qquad\textbf{(B)}\ 97\qquad\textbf{(C)}\ 127\qquad\textbf{(D)}\ 159\qquad\textbf{(E)}\ 729</math>
  
 
+
== Solution 1 ==
==Solution 1==
 
  
 
Notice this is the term <math>a_6</math> in a recursive sequence, defined recursively as <math>a_1 = 3, a_n = 2a_{n-1} + 1.</math> Thus:
 
Notice this is the term <math>a_6</math> in a recursive sequence, defined recursively as <math>a_1 = 3, a_n = 2a_{n-1} + 1.</math> Thus:
 
<cmath>\begin{split}
 
<cmath>\begin{split}
a_2 = 3*2 + 1 = 7.\\
+
a_2 = 3 \cdot 2 + 1 = 7.\\
a_3 = 7 *2 + 1 = 15.\\
+
a_3 = 7 \cdot 2 + 1 = 15.\\
a_4 = 15*2 + 1 = 31.\\
+
a_4 = 15 \cdot 2 + 1 = 31.\\
a_5 = 31*2 + 1 = 63.\\
+
a_5 = 31 \cdot 2 + 1 = 63.\\
a_6 = 63*2 + 1 = \boxed{\textbf{(C)}\ 127}
+
a_6 = 63 \cdot 2 + 1 = \boxed{\textbf{(C)}\ 127}
 
\end{split}</cmath>
 
\end{split}</cmath>
  
==Solution 2==
+
Minor LaTeX edits by fasterthanlight
 +
 
 +
== Solution 2 ==
 
Starting to compute the inner expressions, we see the results are <math>1, 3, 7, 15, \ldots</math>. This is always <math>1</math> less than a power of <math>2</math>. The only admissible answer choice by this rule is thus <math>\boxed{\textbf{(C)}\ 127}</math>.
 
Starting to compute the inner expressions, we see the results are <math>1, 3, 7, 15, \ldots</math>. This is always <math>1</math> less than a power of <math>2</math>. The only admissible answer choice by this rule is thus <math>\boxed{\textbf{(C)}\ 127}</math>.
  
==Solution 3==
+
== Solution 3 ==
  
 
Working our way from the innermost parenthesis outwards and directly computing, we have <math>\boxed{\textbf{(C) } 127}</math>.
 
Working our way from the innermost parenthesis outwards and directly computing, we have <math>\boxed{\textbf{(C) } 127}</math>.
  
==Solution 4==
+
== Solution 4 ==
  
 
If you distribute this you get a sum of the powers of <math>2</math>.  The largest power of <math>2</math> in the series is <math>64</math>, so the sum is <math>\boxed{\textbf{(C)}\ 127}</math>.
 
If you distribute this you get a sum of the powers of <math>2</math>.  The largest power of <math>2</math> in the series is <math>64</math>, so the sum is <math>\boxed{\textbf{(C)}\ 127}</math>.
  
==See Also==
+
 
 +
==Solution 5==
 +
<math>(2(2(2(2(2(2+1)+1)+1)+1)+1)+1)</math>
 +
<math>=(2(2(2(2(2(3)+1)+1)+1)+1)+1)</math>
 +
<math>=(2(2(2(2(6+1)+1)+1)+1)+1)</math>
 +
<math>=(2(2(2(2(7)+1)+1)+1)+1)</math>
 +
<math>=(2(2(2(14+1)+1)+1)+1)</math>
 +
<math>=(2(2(2(15)+1)+1)+1)</math>
 +
<math>=(2(2(30+1)+1)+1)</math>
 +
<math>=(2(2(31)+1)+1)</math>
 +
<math>=(2(62+1)+1)</math>
 +
<math>=(2(63)+1)</math>
 +
<math>=(126+1)</math>
 +
<math>=127 \Longrightarrow \boxed{\textbf{(C)}\ 127}</math>.
 +
 
 +
==Solution 6 (quickest)==
 +
Notice that <math>x^6 + x^5 + x^4 + x^3 + x^2 + x + 1 = x (x (x (x (x (x + 1) + 1) + 1) + 1) + 1) + 1</math>. Substituting <math>2</math> for <math>x</math>, we get <cmath>2(2(2(2(2(2+1)+1)+1)+1)+1)+1 = 2^6 + 2^5 + 2^4 + 2^3 + 2^2 + 2 + 1 = 2^7 - 1 \Longrightarrow \boxed{\textbf{(C)}\ 127}</cmath>
 +
 
 +
==Video Solution==
 +
https://youtu.be/str7kmcRMY8
 +
 
 +
https://youtu.be/kA6W8SwjitA
 +
 
 +
~savannahsolver
 +
 
 +
== See Also ==
 +
 
 
{{AMC10 box|year=2017|ab=A|before=First Problem|num-a=2}}
 
{{AMC10 box|year=2017|ab=A|before=First Problem|num-a=2}}
 
{{MAA Notice}}
 
{{MAA Notice}}

Latest revision as of 12:35, 4 September 2022

Problem

What is the value of $(2(2(2(2(2(2+1)+1)+1)+1)+1)+1)$?

$\textbf{(A)}\ 70\qquad\textbf{(B)}\ 97\qquad\textbf{(C)}\ 127\qquad\textbf{(D)}\ 159\qquad\textbf{(E)}\ 729$

Solution 1

Notice this is the term $a_6$ in a recursive sequence, defined recursively as $a_1 = 3, a_n = 2a_{n-1} + 1.$ Thus: \[\begin{split} a_2 = 3 \cdot 2 + 1 = 7.\\ a_3 = 7 \cdot 2 + 1 = 15.\\ a_4 = 15 \cdot 2 + 1 = 31.\\ a_5 = 31 \cdot 2 + 1 = 63.\\ a_6 = 63 \cdot 2 + 1 = \boxed{\textbf{(C)}\ 127} \end{split}\]

Minor LaTeX edits by fasterthanlight

Solution 2

Starting to compute the inner expressions, we see the results are $1, 3, 7, 15, \ldots$. This is always $1$ less than a power of $2$. The only admissible answer choice by this rule is thus $\boxed{\textbf{(C)}\ 127}$.

Solution 3

Working our way from the innermost parenthesis outwards and directly computing, we have $\boxed{\textbf{(C) } 127}$.

Solution 4

If you distribute this you get a sum of the powers of $2$. The largest power of $2$ in the series is $64$, so the sum is $\boxed{\textbf{(C)}\ 127}$.


Solution 5

$(2(2(2(2(2(2+1)+1)+1)+1)+1)+1)$ $=(2(2(2(2(2(3)+1)+1)+1)+1)+1)$ $=(2(2(2(2(6+1)+1)+1)+1)+1)$ $=(2(2(2(2(7)+1)+1)+1)+1)$ $=(2(2(2(14+1)+1)+1)+1)$ $=(2(2(2(15)+1)+1)+1)$ $=(2(2(30+1)+1)+1)$ $=(2(2(31)+1)+1)$ $=(2(62+1)+1)$ $=(2(63)+1)$ $=(126+1)$ $=127 \Longrightarrow \boxed{\textbf{(C)}\ 127}$.

Solution 6 (quickest)

Notice that $x^6 + x^5 + x^4 + x^3 + x^2 + x + 1 = x (x (x (x (x (x + 1) + 1) + 1) + 1) + 1) + 1$. Substituting $2$ for $x$, we get \[2(2(2(2(2(2+1)+1)+1)+1)+1)+1 = 2^6 + 2^5 + 2^4 + 2^3 + 2^2 + 2 + 1 = 2^7 - 1 \Longrightarrow \boxed{\textbf{(C)}\ 127}\]

Video Solution

https://youtu.be/str7kmcRMY8

https://youtu.be/kA6W8SwjitA

~savannahsolver

See Also

2017 AMC 10A (ProblemsAnswer KeyResources)
Preceded by
First Problem
Followed by
Problem 2
1 2 3 4 5 6 7 8 9 10 11 12 13 14 15 16 17 18 19 20 21 22 23 24 25
All AMC 10 Problems and Solutions

The problems on this page are copyrighted by the Mathematical Association of America's American Mathematics Competitions. AMC logo.png